0 Daumen
441 Aufrufe

Könnte mir jemand bei der Umstellung folgender Aufgabe behilflich sein?

$$\left| a _ { n } - 0 \right| = \frac { 1 } { \sqrt { n ^ { 3 } + 1 } + \sqrt { n ^ { 3 } } } < \frac { 1 } { 2 ( \sqrt { n } ) ^ { 3 } } < 10 ^ { - 3 } \Leftrightarrow n > \sqrt [ 3 ] { ( 500 ) ^ { 2 } } \approx 62,996 $$

D. h. falls \( \mathbb { N } \ni n \geq 63 = : N \)

Avatar von

2 Antworten

+1 Daumen
 
Beste Antwort

$$ \frac{1}{2\sqrt{n}^3}<\epsilon |*\sqrt{n}^3,:\epsilon\\\frac{1}{2\epsilon}<\sqrt{n}^3 |(...)^{2/3}\\\frac{1}{(2\epsilon)^{2/3}} < n \\\text{setze nun }\\\epsilon = 10^{-3} $$

Avatar von 37 k
+1 Daumen

Da geht es nicht um das Umstellen, sondern um eine Abschätzung.

Die Summe der beiden Wurzeln im Nenner ist sicherlich größer als

2 mal die kleinere der beiden (denn beide sind positiv), also größer

als 2*√(n^3)  = 2*(√n) ^3  .  und dann 

1 / 2*(√n) ^3   <  1/1000 

<=>     2*(√n) ^3   >  1000 

<=>  (√n) ^3    >  500

<=>   √(n ^3 )   >  500

<=>   n ^3  >  500^2 

<=>   n   > 3.Wurzel von ( 500^2 )

Avatar von 288 k 🚀

Ein anderes Problem?

Stell deine Frage

Willkommen bei der Mathelounge! Stell deine Frage einfach und kostenlos

x
Made by a lovely community